2
$\begingroup$

Let us consider the space of convergent sequences which is denoted by $c$. The space of all sequences $(x_n)\in c$ with $\lim x_n=0$ is also denoted by $c_0$. Clearly $c_0$ is a proper closed subspace of $c$ under the uniform topology.

These two Banach spaces are the same if and only they are the same as two commutative C*-algebras. But $c_0$ is not unital, however $c$ is unital.

Q1. How one may prove this fact just using functional analysis point of view (elementary tools)?

There exists a functional $\phi\in c^*$ with $\phi(1)=1$ and vanishing on $c_0$ where $1$ is the constant sequence $1,1,\cdots$. The functional $\phi$ corresponds a bounded sequence $(t_n)$.

Q2. How could we determine (explicitly) all bounded sequences $(t_n)$ vanishing on $c_0$?

$\endgroup$
4
  • $\begingroup$ $c_0$ is isometrically isomorphic to $c$ as Banach spaces, but the isomorphism does not preserve the $C^*$-algebra structures. Also, one choice for your functional $\phi$ is given by $\phi(x_n)=\lim_{n\to\infty} x_n$. This does not "correspond to a bounded sequence $(t_n)$." $\endgroup$
    – Teri
    May 13, 2018 at 11:51
  • 3
    $\begingroup$ The proof that there is no linear isometry between these two spaces can be found here: Are these two Banach spaces isometrically isomorphic? and Linear isometry between $c_0$ and $c$. $\endgroup$ May 13, 2018 at 12:14
  • 2
    $\begingroup$ "are the same" means nothing (or means everything). $\endgroup$
    – YCor
    May 13, 2018 at 12:29
  • 1
    $\begingroup$ @Martin Sleziak Ahh, my bad, not isometrically isomorphic. $\endgroup$
    – Teri
    May 13, 2018 at 12:35

3 Answers 3

14
$\begingroup$

The (multiplicative) Banach–Mazur distance between $c$ and $c_0$ is exactly 3:

M. Cambern, On mappings of sequence spaces, Studia Math. 30. (1968), 73-77.

Let me take this opportunity to advertise a rather crazy conjecture (due to Pełczyński, I think):

Suppose that two Banach spaces $C(K)$ and $C(L)$ are isomorphic. Is the Banach–Mazur distance between $C(K)$ and $C(L)$ an integer?

$\endgroup$
4
  • 5
    $\begingroup$ I'd never heard of this conjecture and am surprised it hasn't been investigated/publicised more! $\endgroup$
    – Yemon Choi
    May 13, 2018 at 18:53
  • 1
    $\begingroup$ Interesting conjecture, indeed! i wasn't aware of it either. I am inclined to believe that this is true for countable compact $K$ and $L$ (based on some rough calculations i did in the past.) $\endgroup$ May 13, 2018 at 22:11
  • $\begingroup$ Dear Tomasz, could you please provide some references for this conjecture? Also, do you know its current status? It's very interesting! $\endgroup$ Oct 20, 2023 at 12:40
  • $\begingroup$ @Justdroppedin I think it's been recently solved by Gergont and Piasecki and the paper is about to appear. $\endgroup$ Oct 23, 2023 at 22:38
8
$\begingroup$

We may simply prove that $\ c\ $ and $\ c_0\ $ are not isometric.

Indeed, constant sequences $\ (0)\ $ and $\ (2)\ $ belong to $\ c,\ $ and $\ (1)\in c\ $ is the only point in $\ c\ $ which is half-way from both, this is the unique center.

However, it's a simple exercise to show that for arbitrary $\ x\ z\in c_0\ $ such that $\ ||x-z|| =2\ $ there is a continuum of different points $\ y\in c_0\ $ such that $\ ||x-y|| =||y-z|| = 1.$

$\endgroup$
1
  • 2
    $\begingroup$ Very nice argument. $\endgroup$
    – ABB
    May 14, 2018 at 17:16
7
$\begingroup$

Here is a simple proof that the Banach-Mazur distance between the spaces $c$ and $c_0$ is equal to $3$.

First the map $T:c \to c_0$ defined by \begin{align*} T(x)=\left(3x(\omega) , \frac{3}{2}\big(x(1)- x(\omega)\big), \frac{3}{2}\big(x(2)- x(\omega)\big),\ldots \right) \end{align*} is an isomorphism with $\|T\|\|T^{-1}\|\le 3$.

To show that we can't do better, suppose there is an isomorphism $T:c \to c_0$ with $\|T^{-1}\|\le 1$ and $\|T\|\le K<3$. Let $e_0=(1,1,1,\ldots)$, and $(e_i)_{i\ge 1}$ be the standard unit vectors. Let $\varepsilon=\frac{3-K}{2}$. Let $N$ be such that $|Te_0(t)|<\varepsilon$ for all $t>N$. By pigeonhole principle there exists (in fact, infinitely many) $i_0$ such that $|T(e_{i_0})(t)|<\varepsilon/2$ for all $t\le N$. Consider the vector $T(e_0+2e_{i_0})$. Since $\|e_0+2e_{i_0}\|=3$ we have $\|T(e_0+2e_{i_0})\|\ge 3$. The norm must be 'attained' somewhere so let's check if it is attained at some $t\le N$. $$3\le |T(e_0+2e_{i_0})(t)|\le |T(e_0-e_{i_0})(t)|+3|T(e_{i_0}(t))|<|T(e_0-e_{i_0})(t)|+3\varepsilon/2$$ So $|T(e_0-e_{i_0})(t)|>3-3\varepsilon/2$. But this is impossible since $|T(e_0-e_{i_0})(t)|\le K$ (check that $K>3-\frac{3\varepsilon}{2}$ not possible for $K<3$).

On the other hand, if the norm is attained at some $t>N$ we have $$3\le |T(e_0+2e_{i_0})(t)|<\varepsilon +2|T(e_{i_0})(t)|$$ so $$|T(e_{i_0})(t)|>\frac{3-\varepsilon}{2}$$ But since $\|e_0-2e_{i_0}\|=1$ we have $$K\ge \|T(e_0-2e_{i_0})\|\ge 2|T(e_{i_0})(t)-|T(e_0(t))|\ge 2\frac{3-\varepsilon}{2}-\varepsilon=3-2\varepsilon$$ again leads to a contradiction

$\endgroup$

Your Answer

By clicking “Post Your Answer”, you agree to our terms of service and acknowledge you have read our privacy policy.

Not the answer you're looking for? Browse other questions tagged or ask your own question.